首页 诗词 字典 板报 句子 名言 友答 励志 学校 网站地图
当前位置: 首页 > 外语考试 > GMAT > 综合辅导 >

GMAT逻辑练习题4(附答案)(3)

2012-07-30 
GMAT逻辑练习题

  答案解析

  Prep2012-Pack1-CR-016 VCR007831 Easy

  Reasoning

  What information would provide the strongest evidence about whether introducing more innovative, eye-catching clothing would help the company Increase sales? The company is struggling to keep consumers’ attention because the number of competing clothing brands is increasing faster than demand for clothing. Although innovative, eye-catching clothing would probably attract attention, the plan's ultimate goal is to increase sales. Therefore, look for an answer option that addresses the issue of whether more innovative, eye-catching clothing would actualy increase the company's sales.

  A. The company's plan might or might not work regardless of whether competing companies also frequently introduce new lines of clothing; it might depend on whether those competing fines are also innovative and eye-catching.

  B. As explained above, the question is not whether innovative, eye-catching clothing will attract attention, but rather whether it will increase sales.

  C. This does not speak to whether the company's plan can meet the challenge of the rapidly-changing marketplace

  by introducing more innovative, eye-catching clothing.

  D. Correct. This addresses the issue of whether innovative, eye-catching clothing would actually increase the company's sales.

  E. Knowing whether the company currently sells any innovative, eye-catching clothing does not indicate whether introducing more such clothing would increase or decrease sales.

  The correct answer is D.

  Prep2012-Pack1-CR-017 VCR007918 Easy

  Reasoning

  What would suggest that the reason people in Wistar live longer is not the extra exercise they get from walking in mountainous terrain? The argument is that exercise increases longevity, and people in Wistar have to exercise just to get around in the mountainous terrain. Consequently, that extra exercise must explain their unusual longevity. The argument would be weakened if some other plausible factor could explain their greater longevity; if people in other parts of Bellaria actually get just as much exercise as people in Wistar; or if many people in Wistar have not [wed there long enough to accrue significant benefits from mountain exercise. Therefore, look for an answer option that undermines the argument in one of these ways.

  A. Since we do not know whether people have higher or lower personal incomes in Wistar than elsewhere in Bellaria, the relevance of this information is unclear.

  B. By out the possibility that diet could explain the greater longevity of people in Wistar, this supports the arguments conclusion that mountain exercise is the explanation.

  C. Correct. People who moved to Wistar in middle age or upon retirement have not pursued a typically Wistarian lifestyle for a major part of their fives, even if they five to be near 100. This indicates that factors other than the Wisterian lifestyle may well have the greatest impact on longevity among that substantial group. This group may have greater longevity than among the Wistarian population in general, rendering hazardous any generalizations about the causes of longevity in Wistar.

  D. This indicates that Wistar offers many opportunities for mountain exercise--which other Bellarians seem to get only during vacation. Therefore, this option supports the argument's conclusion that such exercise accounts for the greater longevity of people living in Wistar.

  E. The argument is that people living in Wistar five longer just because of the exercise they get walking in the mountainous terrain as part of their daily routines; they do not have to spend money on recreational activities to get this exercise.

  The correct answer is C.

  Prep2012-Pack1-CR-018 VCR014017 Easy

  Reasoning

  In order to adequately support the arguments conclusion, what piece of information must be an implicit part of the argument? The conclusion of the argument is residents’ complaints about night flights that disturb their sleep are unreasonable. The officials point is that if the residents chose to move into an area with noisy night flights, they cannot reasonably complain about those flights. But this presupposes that the area already experienced noise from night fights when the residents moved there; that the residents knew or should have known about the noise when they bought their homes; and that they could have chosen to live elsewhere. Therefore, look for an answer option that expresses or follows from one or more of these assumptions.

  A. The point could make sense even if the frequency of night flights hasn't changed.

  B. Correct. As explained above, the argument presupposes that the night flights were already happening when the complaining residents first moved to the area; since the night flights started twenty years ago, the argument assumes the residents moved there less than twenty years ago.

  C. Even if the residents acknowledged the benefits they gain from the airport, their complaints about the night flights might still be unreasonable.

  D. Even if the airport could scrape by without night flights, the residents’ complaints about the night flights might still be unreasonable.

  E. Without assuming this, the argument would still be successful--for example, even if it included the information that no homebuyers who can avoid buying houses near the airport do so.

  The correct answer is B.

热点排行